Allowed Publications
Slot System
Featured Buckets
Featured Buckets Admin

Boy Has Had “Bald Spot” Since Birth

Article Type
Changed
Tue, 12/13/2016 - 12:08
Display Headline
Boy Has Had “Bald Spot” Since Birth

ANSWER
The answer is temporal triangular alopecia (choice “d”), an unusual form of permanent hair loss preferentially affecting the exact area depicted in this case.

Alopecia areata (choice “a”) involves localized hair loss. By contrast, this patient never had hair in this area to lose.

Nevus sebaceous (choice “b”) is a congenital hamartoma that is typically hairless; there are no follicles, and the bumpy, rough surface is composed of sebaceous globules.

Cutis aplasia (choice “c”) manifests with hairless lesions, but there is marked aplasia of the skin as well and no surface adnexae, let alone hairs or follicles.

DISCUSSION
Temporal triangular alopecia (TTA) is an unusual type of alopecia. Of unknown origin, it usually affects this area of the scalp—and usually unilaterally. Approximately one-third of TTA patients are born with the condition; the rest develop it in the first two to three years of life. As in this case, it is often wrongly attributed to the use of forceps but has nothing to do with trauma. One school of thought holds that TTA is probably an inherited condition—but others disagree.

TTA was originally known as congenital triangular alopecia. However, when enough cases had been accumulated to accurately determine the nature of the condition, it was realized that TTA is not always congenital or triangular. Thus, a new name was bestowed.

The hallmark of TTA is the normal number of hair follicles that only grow vellus hairs. The solitary peripheral tuft of terminal dark hairs is typical of TTA and thus a confirmatory finding.

TREATMENT/PROGNOSIS
TTA is by definition permanent. Since there’s no inflammation (a key difference from alopecia areata), steroids are useless. The only successful treatment for TTA, if any is attempted, is hair transplantation. As of this writing, the family is mulling this treatment option.

References

Article PDF
Author and Disclosure Information

Joe R. Monroe, MPAS, PA

Issue
Clinician Reviews - 25(3)
Publications
Topics
Page Number
12,14
Legacy Keywords
bald spot, alopecia areata, nevus sebaceous, cutis aplasia, temporal triangular alopecia, TTA, hair loss
Sections
Author and Disclosure Information

Joe R. Monroe, MPAS, PA

Author and Disclosure Information

Joe R. Monroe, MPAS, PA

Article PDF
Article PDF
Related Articles

ANSWER
The answer is temporal triangular alopecia (choice “d”), an unusual form of permanent hair loss preferentially affecting the exact area depicted in this case.

Alopecia areata (choice “a”) involves localized hair loss. By contrast, this patient never had hair in this area to lose.

Nevus sebaceous (choice “b”) is a congenital hamartoma that is typically hairless; there are no follicles, and the bumpy, rough surface is composed of sebaceous globules.

Cutis aplasia (choice “c”) manifests with hairless lesions, but there is marked aplasia of the skin as well and no surface adnexae, let alone hairs or follicles.

DISCUSSION
Temporal triangular alopecia (TTA) is an unusual type of alopecia. Of unknown origin, it usually affects this area of the scalp—and usually unilaterally. Approximately one-third of TTA patients are born with the condition; the rest develop it in the first two to three years of life. As in this case, it is often wrongly attributed to the use of forceps but has nothing to do with trauma. One school of thought holds that TTA is probably an inherited condition—but others disagree.

TTA was originally known as congenital triangular alopecia. However, when enough cases had been accumulated to accurately determine the nature of the condition, it was realized that TTA is not always congenital or triangular. Thus, a new name was bestowed.

The hallmark of TTA is the normal number of hair follicles that only grow vellus hairs. The solitary peripheral tuft of terminal dark hairs is typical of TTA and thus a confirmatory finding.

TREATMENT/PROGNOSIS
TTA is by definition permanent. Since there’s no inflammation (a key difference from alopecia areata), steroids are useless. The only successful treatment for TTA, if any is attempted, is hair transplantation. As of this writing, the family is mulling this treatment option.

ANSWER
The answer is temporal triangular alopecia (choice “d”), an unusual form of permanent hair loss preferentially affecting the exact area depicted in this case.

Alopecia areata (choice “a”) involves localized hair loss. By contrast, this patient never had hair in this area to lose.

Nevus sebaceous (choice “b”) is a congenital hamartoma that is typically hairless; there are no follicles, and the bumpy, rough surface is composed of sebaceous globules.

Cutis aplasia (choice “c”) manifests with hairless lesions, but there is marked aplasia of the skin as well and no surface adnexae, let alone hairs or follicles.

DISCUSSION
Temporal triangular alopecia (TTA) is an unusual type of alopecia. Of unknown origin, it usually affects this area of the scalp—and usually unilaterally. Approximately one-third of TTA patients are born with the condition; the rest develop it in the first two to three years of life. As in this case, it is often wrongly attributed to the use of forceps but has nothing to do with trauma. One school of thought holds that TTA is probably an inherited condition—but others disagree.

TTA was originally known as congenital triangular alopecia. However, when enough cases had been accumulated to accurately determine the nature of the condition, it was realized that TTA is not always congenital or triangular. Thus, a new name was bestowed.

The hallmark of TTA is the normal number of hair follicles that only grow vellus hairs. The solitary peripheral tuft of terminal dark hairs is typical of TTA and thus a confirmatory finding.

TREATMENT/PROGNOSIS
TTA is by definition permanent. Since there’s no inflammation (a key difference from alopecia areata), steroids are useless. The only successful treatment for TTA, if any is attempted, is hair transplantation. As of this writing, the family is mulling this treatment option.

References

References

Issue
Clinician Reviews - 25(3)
Issue
Clinician Reviews - 25(3)
Page Number
12,14
Page Number
12,14
Publications
Publications
Topics
Article Type
Display Headline
Boy Has Had “Bald Spot” Since Birth
Display Headline
Boy Has Had “Bald Spot” Since Birth
Legacy Keywords
bald spot, alopecia areata, nevus sebaceous, cutis aplasia, temporal triangular alopecia, TTA, hair loss
Legacy Keywords
bald spot, alopecia areata, nevus sebaceous, cutis aplasia, temporal triangular alopecia, TTA, hair loss
Sections
Questionnaire Body
Since birth, this 8-year-old boy has had a “bald spot” on his scalp.
What is the cause of this oddly shaped "bald spot"?

Since birth, this 8-year-old boy has had a “bald spot” on his scalp. The pediatrician who attended the birth suggested trauma as the cause, since forceps were used to facilitate delivery. But the problem has failed to resolve, leaving the boy an object of ridicule among his classmates. According to the patient’s ­parents, there has never been any broken skin or hair growth in the area. There is no family history of similar problems, and the child’s health history is unremarkable. The child’s current pediatrician, who made the referral to dermatology, suggested the lesion might be a form of nevus sebaceous. The affected site is roughly triangular, measures about 3 cm on each side, and is located just inside the temporal scalp. The hair loss in this sharply circumscribed area is almost complete, with a lone tuft of darker terminal hairs on the inferior aspect of the site. No redness or epidermal disturbance (eg, scaling) is noted. Dermatoscopic examination (with 10x magnification) reveals a normal number of follicles and hairs. The latter are vellus hairs, except for the aforementioned solitary tuft. The rest of the scalp, including the same location on the opposite side, is free of any significant changes.
Article Source

PURLs Copyright

Inside the Article

Article PDF Media

Lesions’ Pattern Helps Line Up Diagnosis

Article Type
Changed
Tue, 12/13/2016 - 12:08
Display Headline
Lesions’ Pattern Helps Line Up Diagnosis

ANSWER
The correct answer is lichen nitidus (choice “b”), a harmless, self-limited condition of unknown origin. The lesions’ flat-topped (planar) surfaces and tendency to form in linear configurations along lines of trauma (so-called Koebner phenomenon) are also features seen in lichen planus (choice “d”) lesions; however, the latter are almost always pruritic and purple in color. Ironically, the histologic pattern seen in both is almost identical.

An extremely common condition, molluscum contagiosum (choice “a”) presents with multiple tiny papules. But these are not planar, and most will have an umbilicated center. See the Discussion for ways to distinguish it from lichen nitidus.

Flat warts (choice “c”), known as verruca plana, can strongly resemble lichen nitidus, but they are not as flat-topped and do not appear white. They do Koebnerize, however, which occasionally makes the distinction difficult.

DISCUSSION
Lichen nitidus (LN) is an unusual but benign condition primarily affecting children and young adults. Due to the contrast, the white planar papules are easier to see on darker skin. As is the case with many dermatologic diagnoses, LN is easily identified if you’ve heard of it and therefore know what to expect—but much more difficult if you haven’t.

LN’s unique manifestation distinguishes it from other items in the differential. For example, molluscum and LN can easily be confused, especially since both primarily affect children. But the pathognomic central umbilication of molluscum lesions is the key distinguishing feature; the best way to highlight it is with a short blast of liquid nitrogen. (Usually, though, the planar surfaces of LN are sufficient to distinguish it from other conditions.)

In the United States, the term Koebner phenomenon refers to the tendency for lesions to form along areas of trauma, usually in a linear configuration. All four items in our differential can present in this way. However, the term auto-inoculation might be more properly applied to conditions such as warts and molluscum, since the trauma has merely inoculated the organism into the skin. Inflammatory conditions such as LN and lichen planus are not truly “spread” by the trauma.

Linearly configured lesions are sufficiently unusual in dermatology to warrant their own differential. Among those that can present in this manner are psoriasis, lichen sclerosus et atrophicus, and vitiligo.

TREATMENT/PROGNOSIS
Our LN patient did not require any treatment, nor was any possible. The condition is quite likely to clear on its own, leaving little if any evidence in its wake.

I often show affected patients and/or their parents pictures of these types of conditions from our textbooks, for added reassurance. And in this day and age, I direct them to websites where they can do more investigation on their own time.

The effective practice of dermatology (and of all medicine, for that matter) includes more than merely making a correct diagnosis: I believe we’re obliged to “sell” it as well.

References

Article PDF
Author and Disclosure Information

Joe R. Monroe, MPAS, PA

Issue
Clinician Reviews - 25(2)
Publications
Topics
Page Number
10-11
Legacy Keywords
dermatology, lichen nitidus, lesion, warts, lichen planus
Sections
Author and Disclosure Information

Joe R. Monroe, MPAS, PA

Author and Disclosure Information

Joe R. Monroe, MPAS, PA

Article PDF
Article PDF
Related Articles

ANSWER
The correct answer is lichen nitidus (choice “b”), a harmless, self-limited condition of unknown origin. The lesions’ flat-topped (planar) surfaces and tendency to form in linear configurations along lines of trauma (so-called Koebner phenomenon) are also features seen in lichen planus (choice “d”) lesions; however, the latter are almost always pruritic and purple in color. Ironically, the histologic pattern seen in both is almost identical.

An extremely common condition, molluscum contagiosum (choice “a”) presents with multiple tiny papules. But these are not planar, and most will have an umbilicated center. See the Discussion for ways to distinguish it from lichen nitidus.

Flat warts (choice “c”), known as verruca plana, can strongly resemble lichen nitidus, but they are not as flat-topped and do not appear white. They do Koebnerize, however, which occasionally makes the distinction difficult.

DISCUSSION
Lichen nitidus (LN) is an unusual but benign condition primarily affecting children and young adults. Due to the contrast, the white planar papules are easier to see on darker skin. As is the case with many dermatologic diagnoses, LN is easily identified if you’ve heard of it and therefore know what to expect—but much more difficult if you haven’t.

LN’s unique manifestation distinguishes it from other items in the differential. For example, molluscum and LN can easily be confused, especially since both primarily affect children. But the pathognomic central umbilication of molluscum lesions is the key distinguishing feature; the best way to highlight it is with a short blast of liquid nitrogen. (Usually, though, the planar surfaces of LN are sufficient to distinguish it from other conditions.)

In the United States, the term Koebner phenomenon refers to the tendency for lesions to form along areas of trauma, usually in a linear configuration. All four items in our differential can present in this way. However, the term auto-inoculation might be more properly applied to conditions such as warts and molluscum, since the trauma has merely inoculated the organism into the skin. Inflammatory conditions such as LN and lichen planus are not truly “spread” by the trauma.

Linearly configured lesions are sufficiently unusual in dermatology to warrant their own differential. Among those that can present in this manner are psoriasis, lichen sclerosus et atrophicus, and vitiligo.

TREATMENT/PROGNOSIS
Our LN patient did not require any treatment, nor was any possible. The condition is quite likely to clear on its own, leaving little if any evidence in its wake.

I often show affected patients and/or their parents pictures of these types of conditions from our textbooks, for added reassurance. And in this day and age, I direct them to websites where they can do more investigation on their own time.

The effective practice of dermatology (and of all medicine, for that matter) includes more than merely making a correct diagnosis: I believe we’re obliged to “sell” it as well.

ANSWER
The correct answer is lichen nitidus (choice “b”), a harmless, self-limited condition of unknown origin. The lesions’ flat-topped (planar) surfaces and tendency to form in linear configurations along lines of trauma (so-called Koebner phenomenon) are also features seen in lichen planus (choice “d”) lesions; however, the latter are almost always pruritic and purple in color. Ironically, the histologic pattern seen in both is almost identical.

An extremely common condition, molluscum contagiosum (choice “a”) presents with multiple tiny papules. But these are not planar, and most will have an umbilicated center. See the Discussion for ways to distinguish it from lichen nitidus.

Flat warts (choice “c”), known as verruca plana, can strongly resemble lichen nitidus, but they are not as flat-topped and do not appear white. They do Koebnerize, however, which occasionally makes the distinction difficult.

DISCUSSION
Lichen nitidus (LN) is an unusual but benign condition primarily affecting children and young adults. Due to the contrast, the white planar papules are easier to see on darker skin. As is the case with many dermatologic diagnoses, LN is easily identified if you’ve heard of it and therefore know what to expect—but much more difficult if you haven’t.

LN’s unique manifestation distinguishes it from other items in the differential. For example, molluscum and LN can easily be confused, especially since both primarily affect children. But the pathognomic central umbilication of molluscum lesions is the key distinguishing feature; the best way to highlight it is with a short blast of liquid nitrogen. (Usually, though, the planar surfaces of LN are sufficient to distinguish it from other conditions.)

In the United States, the term Koebner phenomenon refers to the tendency for lesions to form along areas of trauma, usually in a linear configuration. All four items in our differential can present in this way. However, the term auto-inoculation might be more properly applied to conditions such as warts and molluscum, since the trauma has merely inoculated the organism into the skin. Inflammatory conditions such as LN and lichen planus are not truly “spread” by the trauma.

Linearly configured lesions are sufficiently unusual in dermatology to warrant their own differential. Among those that can present in this manner are psoriasis, lichen sclerosus et atrophicus, and vitiligo.

TREATMENT/PROGNOSIS
Our LN patient did not require any treatment, nor was any possible. The condition is quite likely to clear on its own, leaving little if any evidence in its wake.

I often show affected patients and/or their parents pictures of these types of conditions from our textbooks, for added reassurance. And in this day and age, I direct them to websites where they can do more investigation on their own time.

The effective practice of dermatology (and of all medicine, for that matter) includes more than merely making a correct diagnosis: I believe we’re obliged to “sell” it as well.

References

References

Issue
Clinician Reviews - 25(2)
Issue
Clinician Reviews - 25(2)
Page Number
10-11
Page Number
10-11
Publications
Publications
Topics
Article Type
Display Headline
Lesions’ Pattern Helps Line Up Diagnosis
Display Headline
Lesions’ Pattern Helps Line Up Diagnosis
Legacy Keywords
dermatology, lichen nitidus, lesion, warts, lichen planus
Legacy Keywords
dermatology, lichen nitidus, lesion, warts, lichen planus
Sections
Questionnaire Body
Six months ago, a 6-year-old boy developed asymptomatic lesions on his elbows, then his knees.
The boy’s mother became alarmed when the lesions started to form in long lines on his arms.

Six months ago, a 6-year-old boy developed asymptomatic lesions on his elbows, then his knees. They slowly spread to other areas, including his forearms. One primary care provider diagnosed probable warts; another, molluscum. The prescribed treatments—liquid nitrogen and tretinoin, respectively—had no effect. The boy’s mother became alarmed when the lesions started to form in long lines on his arms. At that point, she decided to bring him to dermatology for evaluation. Aside from his skin condition, the child is healthy, according to both his mother and the records provided by his primary care provider’s office. The lesions are particularly numerous over the extensor surfaces of the legs—especially the knees—but are also seen on the extensor forearms and elbows. The lesions are exquisitely discrete, identical, tiny white pinpoint papules, all with flat tops. None are umbilicated. In several areas of the arms, linear collections of lesions, some extending as long as 6 cm, are noted. The rest of his exposed type V skin is unremarkable.
Article Source

PURLs Copyright

Inside the Article

Article PDF Media

Seven Years of Pain Between the Toes

Article Type
Changed
Tue, 12/13/2016 - 12:08
Display Headline
Seven Years of Pain Between the Toes

ANSWER
The correct answer is soft corn (choice “c”). They are caused by bony friction and almost always found between the fourth and fifth toes.

Soft corns are often mistaken for warts (choice “a”). But warts don’t present as painful, macerated lesions between the toes.

Morton neuroma (choice “b”) is actually a neurofibroma, not a virtual tumor. It is usually found on the plantar forefoot between the second and third toes.

Interdigital fungal infections (choice “d”) often develop between the fourth and fifth toes and are often macerated. However, they do not take the form of lesions and do not hurt.

DISCUSSION
Soft corns are known in podiatric circles as heloma molle but are sometimes called kissing corns because they’re caused by friction between bony prominences on the fourth and fifth phalanges, which rub together with every step. Normally, these toes are hourglass shaped, but in patients prone to develop soft corns, the proximal bases of the toes are too wide. The type of shoe the patient wears can be an important factor as well, especially when high heels and/or narrow toe boxes are involved.

The treatment of soft corns can be nonsurgical—sometimes as simple as separating the toes with a tuft of lambswool. However, surgical intervention is often required. In such cases, the head of the proximal phalanx is cut and removed to make the adjacent bones more parallel. Occasionally, the skin is so damaged that it too must be removed and the toes sewn together.

Removing corns with chemicals, shaving, or excision provides no lasting relief, since these  methods do not address the underlying structural issues.

Hard corns, also known as heloma durum, tend to develop on the dorsal aspect of the fifth toe secondary to pressure from shoes. Changing the type of shoe worn is one solution, but often, as with soft corns, the underlying bony prominence must be addressed.

There is a third type of corn, the periungual corn, which develops on or near the edge of a nail. These corns are often erroneously called warts.

This patient was referred to a podiatrist, who will likely solve the problem. There is no topical product that can help, and nonsurgical approaches will provide temporary relief at best.

Article PDF
Author and Disclosure Information

 

Joe R. Monroe, MPAS, PA

Joe R. Monroe, MPAS, PA, ­practices at Dawkins ­Dermatology Clinic in Oklahoma City. He is also the founder of the Society of ­Dermatology ­Physician ­Assistants.

Issue
Clinician Reviews - 25(1)
Publications
Topics
Page Number
14-15
Legacy Keywords
dermatology, soft corn
Sections
Author and Disclosure Information

 

Joe R. Monroe, MPAS, PA

Joe R. Monroe, MPAS, PA, ­practices at Dawkins ­Dermatology Clinic in Oklahoma City. He is also the founder of the Society of ­Dermatology ­Physician ­Assistants.

Author and Disclosure Information

 

Joe R. Monroe, MPAS, PA

Joe R. Monroe, MPAS, PA, ­practices at Dawkins ­Dermatology Clinic in Oklahoma City. He is also the founder of the Society of ­Dermatology ­Physician ­Assistants.

Article PDF
Article PDF
Related Articles

ANSWER
The correct answer is soft corn (choice “c”). They are caused by bony friction and almost always found between the fourth and fifth toes.

Soft corns are often mistaken for warts (choice “a”). But warts don’t present as painful, macerated lesions between the toes.

Morton neuroma (choice “b”) is actually a neurofibroma, not a virtual tumor. It is usually found on the plantar forefoot between the second and third toes.

Interdigital fungal infections (choice “d”) often develop between the fourth and fifth toes and are often macerated. However, they do not take the form of lesions and do not hurt.

DISCUSSION
Soft corns are known in podiatric circles as heloma molle but are sometimes called kissing corns because they’re caused by friction between bony prominences on the fourth and fifth phalanges, which rub together with every step. Normally, these toes are hourglass shaped, but in patients prone to develop soft corns, the proximal bases of the toes are too wide. The type of shoe the patient wears can be an important factor as well, especially when high heels and/or narrow toe boxes are involved.

The treatment of soft corns can be nonsurgical—sometimes as simple as separating the toes with a tuft of lambswool. However, surgical intervention is often required. In such cases, the head of the proximal phalanx is cut and removed to make the adjacent bones more parallel. Occasionally, the skin is so damaged that it too must be removed and the toes sewn together.

Removing corns with chemicals, shaving, or excision provides no lasting relief, since these  methods do not address the underlying structural issues.

Hard corns, also known as heloma durum, tend to develop on the dorsal aspect of the fifth toe secondary to pressure from shoes. Changing the type of shoe worn is one solution, but often, as with soft corns, the underlying bony prominence must be addressed.

There is a third type of corn, the periungual corn, which develops on or near the edge of a nail. These corns are often erroneously called warts.

This patient was referred to a podiatrist, who will likely solve the problem. There is no topical product that can help, and nonsurgical approaches will provide temporary relief at best.

ANSWER
The correct answer is soft corn (choice “c”). They are caused by bony friction and almost always found between the fourth and fifth toes.

Soft corns are often mistaken for warts (choice “a”). But warts don’t present as painful, macerated lesions between the toes.

Morton neuroma (choice “b”) is actually a neurofibroma, not a virtual tumor. It is usually found on the plantar forefoot between the second and third toes.

Interdigital fungal infections (choice “d”) often develop between the fourth and fifth toes and are often macerated. However, they do not take the form of lesions and do not hurt.

DISCUSSION
Soft corns are known in podiatric circles as heloma molle but are sometimes called kissing corns because they’re caused by friction between bony prominences on the fourth and fifth phalanges, which rub together with every step. Normally, these toes are hourglass shaped, but in patients prone to develop soft corns, the proximal bases of the toes are too wide. The type of shoe the patient wears can be an important factor as well, especially when high heels and/or narrow toe boxes are involved.

The treatment of soft corns can be nonsurgical—sometimes as simple as separating the toes with a tuft of lambswool. However, surgical intervention is often required. In such cases, the head of the proximal phalanx is cut and removed to make the adjacent bones more parallel. Occasionally, the skin is so damaged that it too must be removed and the toes sewn together.

Removing corns with chemicals, shaving, or excision provides no lasting relief, since these  methods do not address the underlying structural issues.

Hard corns, also known as heloma durum, tend to develop on the dorsal aspect of the fifth toe secondary to pressure from shoes. Changing the type of shoe worn is one solution, but often, as with soft corns, the underlying bony prominence must be addressed.

There is a third type of corn, the periungual corn, which develops on or near the edge of a nail. These corns are often erroneously called warts.

This patient was referred to a podiatrist, who will likely solve the problem. There is no topical product that can help, and nonsurgical approaches will provide temporary relief at best.

Issue
Clinician Reviews - 25(1)
Issue
Clinician Reviews - 25(1)
Page Number
14-15
Page Number
14-15
Publications
Publications
Topics
Article Type
Display Headline
Seven Years of Pain Between the Toes
Display Headline
Seven Years of Pain Between the Toes
Legacy Keywords
dermatology, soft corn
Legacy Keywords
dermatology, soft corn
Sections
Questionnaire Body

 

 

For at least seven years, this 40-year-old man has had pain in the area between the fourth and fifth toes on his left foot. During that time, he has consulted clinicians in a number of settings—including urgent care centers and the emergency department—and received “at least 30” prescriptions for oral antibiotics. Given his persistent pain, none of these treatment attempts has helped. He spends a great deal of time on his feet at work, which worsens the pain. The only relief he experiences is when he goes home at night and removes his socks and shoes. Walking barefoot, he reports, results in relatively little discomfort. The patient claims to be in good health otherwise, specifically denying diabetes. He takes no medications regularly. The skin in the lowest point of the webspace between his fourth and fifth toes is focally thickened, white, and macerated, but there is no redness. The area is exquisitely tender to touch. Examination of the rest of his foot is unremarkable.

 

Disallow All Ads
Article PDF Media

Retired Tour Guide Intends to Maintain Her Tan

Article Type
Changed
Tue, 12/13/2016 - 12:08
Display Headline
Retired Tour Guide Intends to Maintain Her Tan

ANSWER
The correct answer is poikiloderma of Civatte (choice “c”), details of which are discussed below.

Poikiloderma vasculare atrophicans (choice “a”) can be an early indication of T-cell lymphoma but would probably not be chronic or confined to sun-exposed skin.

Several forms of lupus (choice “b”) can present with poikilodermatous skin changes, but these would probably not be chronic.

Dermatoheliosis (choice “d”) is the term for the collective effects of overexposure to the sun, of which poikiloderma of Civatte is but one example.

DISCUSSION
The French dermatologist Achille Civatte (1877-1956) first described this particular pattern of sun damage in 1923—about the same time that sunbathing became fashionable among the well-off in the post-WWI era. He noted the distinct combination of telangiectasias, hyperpigmentation, and epidermal atrophy affecting the bilateral neck and lower face, combined with sharply defined sparing of the portion of the anterior neck shaded by the chin. Poikiloderma of Civatte (PC) is extremely common, especially in middle-aged women and, as one might expect, in those with a history of excessive sun exposure over a period of many years.

Though sun-caused, PC is a purely cosmetic issue and does not lead to skin cancer. While it typically causes no symptoms, it does become more obvious with time. The changes are so gradual that others typically notice them before the patient becomes aware.

Transposing these types of skin changes to other locations would make them considerably more worrisome, specifically in the context of possible incipient T-cell lymphoma—one of the very few types of skin cancer that can take years to evolve into frank cancer. But the atrophy, telangiectasias, and discoloration signaling early cutaneous T-cell lymphoma are usually seen in non–sun-exposed skin, particularly in the waistline and groin.

Poikiloderma vasculare atrophicans is only one of several manifestations termed premycotic. This refers to mycosis fungoides, one of the two most common forms of T-cell lymphoma. Serial biopsy, sometimes over the span of several years, is often used to track such changes.

Pulsed light devices and certain types of lasers have been used successfully to treat PC. Our patient, however, declined treatment, declaring her firm intention to maintain “a healthy tan” year-round.

References

Article PDF
Author and Disclosure Information

Joe R. Monroe, MPAS, PA
Joe R. Monroe, MPAS, PA, ­practices at Dawkins ­Dermatology Clinic in Oklahoma City. He is also the founder of the Society of ­Dermatology ­Physician ­Assistants.

Issue
Clinician Reviews - 24(12)
Publications
Topics
Page Number
9-10
Legacy Keywords
dermatology, Poikiloderma of Civatte, neck, pigmentary changes, telangiectasia, hyperpigmentation, epidermal atrophy
Sections
Author and Disclosure Information

Joe R. Monroe, MPAS, PA
Joe R. Monroe, MPAS, PA, ­practices at Dawkins ­Dermatology Clinic in Oklahoma City. He is also the founder of the Society of ­Dermatology ­Physician ­Assistants.

Author and Disclosure Information

Joe R. Monroe, MPAS, PA
Joe R. Monroe, MPAS, PA, ­practices at Dawkins ­Dermatology Clinic in Oklahoma City. He is also the founder of the Society of ­Dermatology ­Physician ­Assistants.

Article PDF
Article PDF
Related Articles

ANSWER
The correct answer is poikiloderma of Civatte (choice “c”), details of which are discussed below.

Poikiloderma vasculare atrophicans (choice “a”) can be an early indication of T-cell lymphoma but would probably not be chronic or confined to sun-exposed skin.

Several forms of lupus (choice “b”) can present with poikilodermatous skin changes, but these would probably not be chronic.

Dermatoheliosis (choice “d”) is the term for the collective effects of overexposure to the sun, of which poikiloderma of Civatte is but one example.

DISCUSSION
The French dermatologist Achille Civatte (1877-1956) first described this particular pattern of sun damage in 1923—about the same time that sunbathing became fashionable among the well-off in the post-WWI era. He noted the distinct combination of telangiectasias, hyperpigmentation, and epidermal atrophy affecting the bilateral neck and lower face, combined with sharply defined sparing of the portion of the anterior neck shaded by the chin. Poikiloderma of Civatte (PC) is extremely common, especially in middle-aged women and, as one might expect, in those with a history of excessive sun exposure over a period of many years.

Though sun-caused, PC is a purely cosmetic issue and does not lead to skin cancer. While it typically causes no symptoms, it does become more obvious with time. The changes are so gradual that others typically notice them before the patient becomes aware.

Transposing these types of skin changes to other locations would make them considerably more worrisome, specifically in the context of possible incipient T-cell lymphoma—one of the very few types of skin cancer that can take years to evolve into frank cancer. But the atrophy, telangiectasias, and discoloration signaling early cutaneous T-cell lymphoma are usually seen in non–sun-exposed skin, particularly in the waistline and groin.

Poikiloderma vasculare atrophicans is only one of several manifestations termed premycotic. This refers to mycosis fungoides, one of the two most common forms of T-cell lymphoma. Serial biopsy, sometimes over the span of several years, is often used to track such changes.

Pulsed light devices and certain types of lasers have been used successfully to treat PC. Our patient, however, declined treatment, declaring her firm intention to maintain “a healthy tan” year-round.

ANSWER
The correct answer is poikiloderma of Civatte (choice “c”), details of which are discussed below.

Poikiloderma vasculare atrophicans (choice “a”) can be an early indication of T-cell lymphoma but would probably not be chronic or confined to sun-exposed skin.

Several forms of lupus (choice “b”) can present with poikilodermatous skin changes, but these would probably not be chronic.

Dermatoheliosis (choice “d”) is the term for the collective effects of overexposure to the sun, of which poikiloderma of Civatte is but one example.

DISCUSSION
The French dermatologist Achille Civatte (1877-1956) first described this particular pattern of sun damage in 1923—about the same time that sunbathing became fashionable among the well-off in the post-WWI era. He noted the distinct combination of telangiectasias, hyperpigmentation, and epidermal atrophy affecting the bilateral neck and lower face, combined with sharply defined sparing of the portion of the anterior neck shaded by the chin. Poikiloderma of Civatte (PC) is extremely common, especially in middle-aged women and, as one might expect, in those with a history of excessive sun exposure over a period of many years.

Though sun-caused, PC is a purely cosmetic issue and does not lead to skin cancer. While it typically causes no symptoms, it does become more obvious with time. The changes are so gradual that others typically notice them before the patient becomes aware.

Transposing these types of skin changes to other locations would make them considerably more worrisome, specifically in the context of possible incipient T-cell lymphoma—one of the very few types of skin cancer that can take years to evolve into frank cancer. But the atrophy, telangiectasias, and discoloration signaling early cutaneous T-cell lymphoma are usually seen in non–sun-exposed skin, particularly in the waistline and groin.

Poikiloderma vasculare atrophicans is only one of several manifestations termed premycotic. This refers to mycosis fungoides, one of the two most common forms of T-cell lymphoma. Serial biopsy, sometimes over the span of several years, is often used to track such changes.

Pulsed light devices and certain types of lasers have been used successfully to treat PC. Our patient, however, declined treatment, declaring her firm intention to maintain “a healthy tan” year-round.

References

References

Issue
Clinician Reviews - 24(12)
Issue
Clinician Reviews - 24(12)
Page Number
9-10
Page Number
9-10
Publications
Publications
Topics
Article Type
Display Headline
Retired Tour Guide Intends to Maintain Her Tan
Display Headline
Retired Tour Guide Intends to Maintain Her Tan
Legacy Keywords
dermatology, Poikiloderma of Civatte, neck, pigmentary changes, telangiectasia, hyperpigmentation, epidermal atrophy
Legacy Keywords
dermatology, Poikiloderma of Civatte, neck, pigmentary changes, telangiectasia, hyperpigmentation, epidermal atrophy
Sections
Questionnaire Body
This woman's acquaintances have noticed odd changes to skin on her neck.
Changes to the skin on this woman's neck have slowly become more noticeable over the years.

A 60-year-old woman is seen for complaints of skin changes on her neck that have slowly become more noticeable over a period of years. Although asymptomatic, these changes have been observed by others, who brought them to the patient’s attention. The patient worked as a tour guide in Arizona for 20 years, leading groups along desert trails to view native flora and fauna. During that time, she maintained a dark tan almost year-round, tanning easily and never using sunscreen. The patient has type III skin, bluish gray eyes, and light brown hair. Dark brown–to–red mottled pigmentary changes are seen on the sides of her neck; the central portion of the anterior neck is sharply spared. On closer inspection, many fine telangiectasias are noted in these same areas, as well as on the sun-exposed areas of the face. Aside from her skin changes, the patient claims to be quite healthy, with no joint pain, fever, or malaise.
Article Source

PURLs Copyright

Inside the Article

Article PDF Media

Is It Ringworm, Herpes— Or Something Else Entirely?

Article Type
Changed
Tue, 12/13/2016 - 12:08
Display Headline
Is It Ringworm, Herpes— Or Something Else Entirely?

ANSWER

The correct answer is impetigo (choice “c”), a superficial infection usually caused by a combination of staph and strep organisms.

Psoriasis (choice “a”) would have presented with white, tenacious scaling and would not have been acute in onset.

Eczema (choice “b”) is definitely possible, but the patient’s rash has features not seen with this condition; see Discussion for details.

Fungal infection (choice “d”) is also definitely in the differential, but it is unlikely given the negative KOH, the lack of any source for such infection, and the complete lack of response to tolnaftate cream.

DISCUSSION

Impetigo has also been called impetiginized dermatitis because it almost always starts with minor breaks in the skin as a result of conditions such as eczema, acne, contact dermatitis, or insect bite. Thus provided with access to deeper portions of the epithelial surface, bacterial organisms that normally cause no problems on intact skin are able to create a minor but annoying condition we have come to call impetigo.

Mistakenly called infantigo in large parts of the United States, impetigo is quite common but nonetheless alarming. Rarely associated with morbidity, it tends to resolve in two to three weeks at most, even without treatment.

Impetigo has the reputation of being highly contagious; given enough heat and humidity, close living conditions, and lack of regular bathing and/or adequate treatment, it can spread rapidly. Those conditions existed commonly 100 years ago, when bathing was sporadic and often cursory, and multiple family members lived and slept in close quarters. In those days before the introduction of antibiotics, there were no good topical antimicrobial agents, either.

Another factor played a major role in impetigo, bolstering its fearsome reputation. The strains of strep (group A b-hemolytic strep) that caused most impetigo in those days included several so-called nephritogenic strains that could lead to a dreaded complication: acute poststreptococcal glomerulonephritis (APSGN). Also called Bright disease, it could and did lead to fatal renal failure—about which little could be done at the time.

Fortunately, such nephritogenic strains of strep are unusual now, with APSGN occurring at a rate of about 1:1,000,000 in developed countries. In those locations, most people live far different lives today, bathing and changing clothes daily and living in much less cramped quarters.

The patient’s atopy likely had an impact, for several reasons: Since staph colonization of atopic persons is quite common, it’s more likely that an infection will develop. Also, thinner skin that is easily broken, a plethora of complicating problems (eg, dry skin, eczema, contact dermatitis, and exaggerated reactions to insect bites), and a lower threshold for itching all make atopic persons more susceptible to infection.

Most likely, our patient had a touch of eczema or dry skin and scratched it. Then, as the condition progressed, she scratched it more. The peroxide she used would have been highly irritating, serving only to worsen matters.

From a diagnostic point of view, the honey-colored crust covering the lesion and the context in which it developed led to a provisional diagnosis of impetiginized dermatitis. She was treated with oral cephalexin (500 mg tid for 7 d), topical mupirocin (applied bid), and topical hydrocortisone cream 2.5% (daily application). At one week’s follow-up, the patient’s skin was almost totally clear. It’s very unlikely she’ll have any residual scarring or blemish.

Had the diagnosis been unclear, or had the patient not responded to treatment, other diagnoses would have been considered. Among them: discoid lupus, psoriasis, contact dermatitis, and Darier disease.

References

Article PDF
Author and Disclosure Information

Joe R. Monroe, MPAS, PA, ­practices at Dawkins ­Dermatology Clinic in Oklahoma City. He is also the founder of the Society of ­Dermatology ­Physician ­Assistants.

Issue
Clinician Reviews - 24(11)
Publications
Topics
Page Number
8-9
Legacy Keywords
dermatology, impetigo, ringworm, herpes, rash, crusty lesions, itchy, acyclovir, tolnaftate
Sections
Author and Disclosure Information

Joe R. Monroe, MPAS, PA, ­practices at Dawkins ­Dermatology Clinic in Oklahoma City. He is also the founder of the Society of ­Dermatology ­Physician ­Assistants.

Author and Disclosure Information

Joe R. Monroe, MPAS, PA, ­practices at Dawkins ­Dermatology Clinic in Oklahoma City. He is also the founder of the Society of ­Dermatology ­Physician ­Assistants.

Article PDF
Article PDF

ANSWER

The correct answer is impetigo (choice “c”), a superficial infection usually caused by a combination of staph and strep organisms.

Psoriasis (choice “a”) would have presented with white, tenacious scaling and would not have been acute in onset.

Eczema (choice “b”) is definitely possible, but the patient’s rash has features not seen with this condition; see Discussion for details.

Fungal infection (choice “d”) is also definitely in the differential, but it is unlikely given the negative KOH, the lack of any source for such infection, and the complete lack of response to tolnaftate cream.

DISCUSSION

Impetigo has also been called impetiginized dermatitis because it almost always starts with minor breaks in the skin as a result of conditions such as eczema, acne, contact dermatitis, or insect bite. Thus provided with access to deeper portions of the epithelial surface, bacterial organisms that normally cause no problems on intact skin are able to create a minor but annoying condition we have come to call impetigo.

Mistakenly called infantigo in large parts of the United States, impetigo is quite common but nonetheless alarming. Rarely associated with morbidity, it tends to resolve in two to three weeks at most, even without treatment.

Impetigo has the reputation of being highly contagious; given enough heat and humidity, close living conditions, and lack of regular bathing and/or adequate treatment, it can spread rapidly. Those conditions existed commonly 100 years ago, when bathing was sporadic and often cursory, and multiple family members lived and slept in close quarters. In those days before the introduction of antibiotics, there were no good topical antimicrobial agents, either.

Another factor played a major role in impetigo, bolstering its fearsome reputation. The strains of strep (group A b-hemolytic strep) that caused most impetigo in those days included several so-called nephritogenic strains that could lead to a dreaded complication: acute poststreptococcal glomerulonephritis (APSGN). Also called Bright disease, it could and did lead to fatal renal failure—about which little could be done at the time.

Fortunately, such nephritogenic strains of strep are unusual now, with APSGN occurring at a rate of about 1:1,000,000 in developed countries. In those locations, most people live far different lives today, bathing and changing clothes daily and living in much less cramped quarters.

The patient’s atopy likely had an impact, for several reasons: Since staph colonization of atopic persons is quite common, it’s more likely that an infection will develop. Also, thinner skin that is easily broken, a plethora of complicating problems (eg, dry skin, eczema, contact dermatitis, and exaggerated reactions to insect bites), and a lower threshold for itching all make atopic persons more susceptible to infection.

Most likely, our patient had a touch of eczema or dry skin and scratched it. Then, as the condition progressed, she scratched it more. The peroxide she used would have been highly irritating, serving only to worsen matters.

From a diagnostic point of view, the honey-colored crust covering the lesion and the context in which it developed led to a provisional diagnosis of impetiginized dermatitis. She was treated with oral cephalexin (500 mg tid for 7 d), topical mupirocin (applied bid), and topical hydrocortisone cream 2.5% (daily application). At one week’s follow-up, the patient’s skin was almost totally clear. It’s very unlikely she’ll have any residual scarring or blemish.

Had the diagnosis been unclear, or had the patient not responded to treatment, other diagnoses would have been considered. Among them: discoid lupus, psoriasis, contact dermatitis, and Darier disease.

ANSWER

The correct answer is impetigo (choice “c”), a superficial infection usually caused by a combination of staph and strep organisms.

Psoriasis (choice “a”) would have presented with white, tenacious scaling and would not have been acute in onset.

Eczema (choice “b”) is definitely possible, but the patient’s rash has features not seen with this condition; see Discussion for details.

Fungal infection (choice “d”) is also definitely in the differential, but it is unlikely given the negative KOH, the lack of any source for such infection, and the complete lack of response to tolnaftate cream.

DISCUSSION

Impetigo has also been called impetiginized dermatitis because it almost always starts with minor breaks in the skin as a result of conditions such as eczema, acne, contact dermatitis, or insect bite. Thus provided with access to deeper portions of the epithelial surface, bacterial organisms that normally cause no problems on intact skin are able to create a minor but annoying condition we have come to call impetigo.

Mistakenly called infantigo in large parts of the United States, impetigo is quite common but nonetheless alarming. Rarely associated with morbidity, it tends to resolve in two to three weeks at most, even without treatment.

Impetigo has the reputation of being highly contagious; given enough heat and humidity, close living conditions, and lack of regular bathing and/or adequate treatment, it can spread rapidly. Those conditions existed commonly 100 years ago, when bathing was sporadic and often cursory, and multiple family members lived and slept in close quarters. In those days before the introduction of antibiotics, there were no good topical antimicrobial agents, either.

Another factor played a major role in impetigo, bolstering its fearsome reputation. The strains of strep (group A b-hemolytic strep) that caused most impetigo in those days included several so-called nephritogenic strains that could lead to a dreaded complication: acute poststreptococcal glomerulonephritis (APSGN). Also called Bright disease, it could and did lead to fatal renal failure—about which little could be done at the time.

Fortunately, such nephritogenic strains of strep are unusual now, with APSGN occurring at a rate of about 1:1,000,000 in developed countries. In those locations, most people live far different lives today, bathing and changing clothes daily and living in much less cramped quarters.

The patient’s atopy likely had an impact, for several reasons: Since staph colonization of atopic persons is quite common, it’s more likely that an infection will develop. Also, thinner skin that is easily broken, a plethora of complicating problems (eg, dry skin, eczema, contact dermatitis, and exaggerated reactions to insect bites), and a lower threshold for itching all make atopic persons more susceptible to infection.

Most likely, our patient had a touch of eczema or dry skin and scratched it. Then, as the condition progressed, she scratched it more. The peroxide she used would have been highly irritating, serving only to worsen matters.

From a diagnostic point of view, the honey-colored crust covering the lesion and the context in which it developed led to a provisional diagnosis of impetiginized dermatitis. She was treated with oral cephalexin (500 mg tid for 7 d), topical mupirocin (applied bid), and topical hydrocortisone cream 2.5% (daily application). At one week’s follow-up, the patient’s skin was almost totally clear. It’s very unlikely she’ll have any residual scarring or blemish.

Had the diagnosis been unclear, or had the patient not responded to treatment, other diagnoses would have been considered. Among them: discoid lupus, psoriasis, contact dermatitis, and Darier disease.

References

References

Issue
Clinician Reviews - 24(11)
Issue
Clinician Reviews - 24(11)
Page Number
8-9
Page Number
8-9
Publications
Publications
Topics
Article Type
Display Headline
Is It Ringworm, Herpes— Or Something Else Entirely?
Display Headline
Is It Ringworm, Herpes— Or Something Else Entirely?
Legacy Keywords
dermatology, impetigo, ringworm, herpes, rash, crusty lesions, itchy, acyclovir, tolnaftate
Legacy Keywords
dermatology, impetigo, ringworm, herpes, rash, crusty lesions, itchy, acyclovir, tolnaftate
Sections
Questionnaire Body

A 16-year-old girl is referred to dermatology by her pediatrician for evaluation of a rash on her face. She is currently taking acyclovir (dose unknown) as prescribed by her pediatrician for presumed herpetic infection. Previous treatment attempts with OTC tolnaftate cream and various OTC moisturizers have failed. The rash manifested several weeks ago with two scaly bumps on her left cheek and temple area, which the patient admits to “picking” at. Initially, the lesions itched a bit, but they became larger and more symptomatic after she applied hydrogen peroxide to them several times. She then began to scrub the lesions vigorously with antibacterial soap while continuing to apply the peroxide. Subsequently, she presented to an urgent care clinic, where she was diagnosed with “ringworm” (and advised to use tolnaftate cream), and then to her pediatrician, with the aforementioned result. Aside from seasonal allergies and periodic episodes of eczema, the patient’s health is excellent. She has no pets. Examination reveals large, annular, honey-colored crusts focally located on the left side of the patient’s face. Faint pinkness is noted peripherally around the lesions. Modest but palpable adenopathy is detected in the pretragal and submental nodal areas. Though symptomatic, the patient is in no distress. A KOH prep taken from the scaly periphery is negative for fungal elements.
Article Source

PURLs Copyright

Inside the Article

Article PDF Media

Postop Patient Reports “Wound Infection”

Article Type
Changed
Tue, 12/13/2016 - 12:08
Display Headline
Postop Patient Reports “Wound Infection”

ANSWER

The correct answer is an allergic reaction to a contactant, most likely the triple-antibiotic ointment (choice “d”).

Irritant reactions to tape adhesive (choice “a”) are extremely common. However, the resultant rash would have been confined to the linear areas where the tape touched his skin.

Dissolving sutures, such as those used in this case, can provoke a “suture granuloma”—essentially a foreign body reaction to the suture material (choice “b”). But this would have caused a focal area of swelling and redness, and very possibly a show of pus.

Postop wound infections (choice “c”) are also quite common. However, they would not manifest solely with itching in a papulovesicular rash surrounding the wound. Had infection developed, the redness would have been broad-based, with ill-defined margins, and the patient’s complaint would have been of pain, not itching. No vesicles would have been seen with bacterial infection.

DISCUSSION

This case illustrates the phenomenon of “treatment as problem,” in which the medication the patient applies becomes more problematic than the condition being addressed. Reactions to the neomycin in triple-antibiotic ointment are common but still provoke considerable worry on the part of patients and providers alike, especially when mistaken for “infection.”

This patient, like many, was dubious of the diagnosis, pointing out that he had used this same topical medication on many occasions without incident (though not recently). What he didn’t know is that it takes repeated exposure to a given allergen to develop T-memory cells that eventually begin to react. This same phenomenon is seen with poison ivy; patients will recall the ability, as a child, to practically wallow in poison ivy with impunity, making them doubtful about being allergic to it as an adult.

Neomycin, an aminoglycoside with a fairly wide spectrum of antibacterial activity, was first noted as a contact allergen in 1952. It is such a notorious offender that it was named Allergen of the Year in 2010 by the American Contact Dermatology Society.

For the past 20 years, 7% to 13% of patch tests surveyed were positive for neomycin. For reasons not entirely clear, Americans older than 60 are 150% more likely to experience a reaction to neomycin than are younger patients. (It could simply be that they’ve had more chances for exposure.)

In another interesting twist, the ointment vehicle appears to play a role. A reaction to this preparation is considerably more likely than to the same drug in other forms (eg, powders, solutions, creams). This is true of most medications, such as topical steroids, which are effectively self-occluded by this vehicle.

Persons with impaired barrier function, such as those with atopic dermatitis or whose skin has been prepped for surgery, appear to be at increased risk for these types of contact dermatoses.

Though there are other items in the differential, the configuration of the papulovesicular rash and the sole symptom of itching are essentially pathognomic for contact dermatitis. Besides the use of potent topical steroids for a few days, the real “cure” for this problem is for the patient to switch to “double-antibiotic” creams or ointments that do not include neomycin. 

Article PDF
Author and Disclosure Information

 

Joe R. Monroe, MPAS, PA, ­practices at Dawkins ­Dermatology Clinic in Oklahoma City. He is also the founder of the Society of ­Dermatology ­Physician ­Assistants.

Issue
Clinician Reviews - 24(10)
Publications
Topics
Page Number
16-17
Legacy Keywords
sharply defined, bright pink, papulovesicular rash, triple-antibiotic ointment, neomycin, contact allergen
Sections
Author and Disclosure Information

 

Joe R. Monroe, MPAS, PA, ­practices at Dawkins ­Dermatology Clinic in Oklahoma City. He is also the founder of the Society of ­Dermatology ­Physician ­Assistants.

Author and Disclosure Information

 

Joe R. Monroe, MPAS, PA, ­practices at Dawkins ­Dermatology Clinic in Oklahoma City. He is also the founder of the Society of ­Dermatology ­Physician ­Assistants.

Article PDF
Article PDF

ANSWER

The correct answer is an allergic reaction to a contactant, most likely the triple-antibiotic ointment (choice “d”).

Irritant reactions to tape adhesive (choice “a”) are extremely common. However, the resultant rash would have been confined to the linear areas where the tape touched his skin.

Dissolving sutures, such as those used in this case, can provoke a “suture granuloma”—essentially a foreign body reaction to the suture material (choice “b”). But this would have caused a focal area of swelling and redness, and very possibly a show of pus.

Postop wound infections (choice “c”) are also quite common. However, they would not manifest solely with itching in a papulovesicular rash surrounding the wound. Had infection developed, the redness would have been broad-based, with ill-defined margins, and the patient’s complaint would have been of pain, not itching. No vesicles would have been seen with bacterial infection.

DISCUSSION

This case illustrates the phenomenon of “treatment as problem,” in which the medication the patient applies becomes more problematic than the condition being addressed. Reactions to the neomycin in triple-antibiotic ointment are common but still provoke considerable worry on the part of patients and providers alike, especially when mistaken for “infection.”

This patient, like many, was dubious of the diagnosis, pointing out that he had used this same topical medication on many occasions without incident (though not recently). What he didn’t know is that it takes repeated exposure to a given allergen to develop T-memory cells that eventually begin to react. This same phenomenon is seen with poison ivy; patients will recall the ability, as a child, to practically wallow in poison ivy with impunity, making them doubtful about being allergic to it as an adult.

Neomycin, an aminoglycoside with a fairly wide spectrum of antibacterial activity, was first noted as a contact allergen in 1952. It is such a notorious offender that it was named Allergen of the Year in 2010 by the American Contact Dermatology Society.

For the past 20 years, 7% to 13% of patch tests surveyed were positive for neomycin. For reasons not entirely clear, Americans older than 60 are 150% more likely to experience a reaction to neomycin than are younger patients. (It could simply be that they’ve had more chances for exposure.)

In another interesting twist, the ointment vehicle appears to play a role. A reaction to this preparation is considerably more likely than to the same drug in other forms (eg, powders, solutions, creams). This is true of most medications, such as topical steroids, which are effectively self-occluded by this vehicle.

Persons with impaired barrier function, such as those with atopic dermatitis or whose skin has been prepped for surgery, appear to be at increased risk for these types of contact dermatoses.

Though there are other items in the differential, the configuration of the papulovesicular rash and the sole symptom of itching are essentially pathognomic for contact dermatitis. Besides the use of potent topical steroids for a few days, the real “cure” for this problem is for the patient to switch to “double-antibiotic” creams or ointments that do not include neomycin. 

ANSWER

The correct answer is an allergic reaction to a contactant, most likely the triple-antibiotic ointment (choice “d”).

Irritant reactions to tape adhesive (choice “a”) are extremely common. However, the resultant rash would have been confined to the linear areas where the tape touched his skin.

Dissolving sutures, such as those used in this case, can provoke a “suture granuloma”—essentially a foreign body reaction to the suture material (choice “b”). But this would have caused a focal area of swelling and redness, and very possibly a show of pus.

Postop wound infections (choice “c”) are also quite common. However, they would not manifest solely with itching in a papulovesicular rash surrounding the wound. Had infection developed, the redness would have been broad-based, with ill-defined margins, and the patient’s complaint would have been of pain, not itching. No vesicles would have been seen with bacterial infection.

DISCUSSION

This case illustrates the phenomenon of “treatment as problem,” in which the medication the patient applies becomes more problematic than the condition being addressed. Reactions to the neomycin in triple-antibiotic ointment are common but still provoke considerable worry on the part of patients and providers alike, especially when mistaken for “infection.”

This patient, like many, was dubious of the diagnosis, pointing out that he had used this same topical medication on many occasions without incident (though not recently). What he didn’t know is that it takes repeated exposure to a given allergen to develop T-memory cells that eventually begin to react. This same phenomenon is seen with poison ivy; patients will recall the ability, as a child, to practically wallow in poison ivy with impunity, making them doubtful about being allergic to it as an adult.

Neomycin, an aminoglycoside with a fairly wide spectrum of antibacterial activity, was first noted as a contact allergen in 1952. It is such a notorious offender that it was named Allergen of the Year in 2010 by the American Contact Dermatology Society.

For the past 20 years, 7% to 13% of patch tests surveyed were positive for neomycin. For reasons not entirely clear, Americans older than 60 are 150% more likely to experience a reaction to neomycin than are younger patients. (It could simply be that they’ve had more chances for exposure.)

In another interesting twist, the ointment vehicle appears to play a role. A reaction to this preparation is considerably more likely than to the same drug in other forms (eg, powders, solutions, creams). This is true of most medications, such as topical steroids, which are effectively self-occluded by this vehicle.

Persons with impaired barrier function, such as those with atopic dermatitis or whose skin has been prepped for surgery, appear to be at increased risk for these types of contact dermatoses.

Though there are other items in the differential, the configuration of the papulovesicular rash and the sole symptom of itching are essentially pathognomic for contact dermatitis. Besides the use of potent topical steroids for a few days, the real “cure” for this problem is for the patient to switch to “double-antibiotic” creams or ointments that do not include neomycin. 

Issue
Clinician Reviews - 24(10)
Issue
Clinician Reviews - 24(10)
Page Number
16-17
Page Number
16-17
Publications
Publications
Topics
Article Type
Display Headline
Postop Patient Reports “Wound Infection”
Display Headline
Postop Patient Reports “Wound Infection”
Legacy Keywords
sharply defined, bright pink, papulovesicular rash, triple-antibiotic ointment, neomycin, contact allergen
Legacy Keywords
sharply defined, bright pink, papulovesicular rash, triple-antibiotic ointment, neomycin, contact allergen
Sections
Questionnaire Body

 

 

A week ago, a 56-year-old man had a skin cancer surgically removed. Last night, he presented to an urgent care clinic for evaluation of a “wound infection” and received a prescription for double-strength trimethoprim/sulfa tablets (to be taken bid for 10 days). He is now in the dermatology office for follow-up. According to the patient, the problem manifested two days postop. There was no associated pain, only itching. The patient feels fine, with no fever or malaise, and there is no history of immunosuppression. He reports following his postop instructions well, changing his bandage daily and using triple-antibiotic ointment to dress the wound directly. The immediate peri-incisional area is indicated as the source of the problem. Surrounding the incision, which is healing well otherwise, is a sharply defined, bright pink, papulovesicular rash on a slightly edematous base. There is no tenderness on palpation, and no purulent material can be expressed from the wound. The area is only slightly warmer than the surrounding skin.

 

Disallow All Ads
Article PDF Media

Itchy Lesion Heralds Pervasive Problem

Article Type
Changed
Tue, 12/13/2016 - 12:08
Display Headline
Itchy Lesion Heralds Pervasive Problem

ANSWER
The correct answer is pityriasis rosea (choice “b”), a common and very distinctive eruption related to human herpesvirus 6 and 7.

Allergic reaction to methotrexate (choice “a”), while far from unknown, does not resemble pityriasis rosea. It also would not be limited to such a relatively small area.

Pityriasis rosea is often designated as “fungal infection” (choice “c”) by the uninitiated. However, the lesions of dermatophytosis would be round, with a leading scaly edge, and unlikely to be found in this distribution.

Secondary syphilis (choice “d”) is a major item in the pityriasis rosea differential, but it almost always involves the palms and soles and the lesions would be round (not oval) scaly brown papules. Furthermore, assuming we have an honest patient, we’re also missing a source for sexually transmitted infection.

DISCUSSION
One could hardly ask for a more classic case of pityriasis rosea (PR), which primarily affects patients ages 14 to 40. Alas, that being said, one cannot depend on seeing all these clues in every PR patient.

For example, the herald patch (also known as the mother patch) is missing in at least half of cases. In others, the lesions are smaller, sparser, and more papular (especially in young black patients). The condition may even be confined to intertriginous areas (eg, the groin and/or axillae); this is known as inverse PR.

While salmon-colored scaly lesions are considered a classic presentation, PR can present with darker ovoid macules that have minimal scale and, rarely, become bullous. Involvement above the neck is rare.

What is consistent and dependable among signs of PR is the centripetal scale, seen even in the smallest lesions. This scale is so fine that the old dermatology texts called it “cigarette paper” scale or “scurf.”

After decades of speculation, researchers finally provided strong evidence of the probable cause  of PR: replication of human herpesvirus 6 and 7, present in mono-
nuclear cells of lesional skin. Though universally acquired in childhood, these viruses are thought to remain latent until reactivated, leading to viremia.

Itching can be moderately severe in a minority of cases. Most patients, such as this one, are not bothered much by the condition once they understand its self-limited nature. They usually are not happy, however, to learn that it could persist for nine weeks or more, whether treated or not.

UV light exposure can be helpful in hastening PR’s departure, and topical corticosteroids (class III or IV; eg, triamcinolone 0.1% cream) can help control the itching. Neither oral nor topical antihistamines will help, since PR is not a histamine-mediated problem.

If the diagnosis is in doubt, a punch biopsy could at least rule out the more serious items in the differential, which include syphilis, drug rash, and psoriasis. In cases in which fungal origin is a possibility, a quick KOH prep will settle the issue. However, it must be remembered that one doesn’t just “get” a fungal infection. There has to be a source (animal, child), and that source is usually identified with minimal history taking.

Article PDF
Author and Disclosure Information

 

Joe R. Monroe, MPAS, PA, ­practices at Dawkins ­Dermatology Clinic in Oklahoma City. He is also the founder of the Society of ­Dermatology ­Physician ­Assistants.

Issue
Clinician Reviews - 24(9)
Publications
Topics
Page Number
11-12
Legacy Keywords
dermatology, dermadiagnosis, neck, chest, rash, allergic reaction, methotrexate, Pityriasis rosea, fungal infection, Secondary syphilis
Sections
Author and Disclosure Information

 

Joe R. Monroe, MPAS, PA, ­practices at Dawkins ­Dermatology Clinic in Oklahoma City. He is also the founder of the Society of ­Dermatology ­Physician ­Assistants.

Author and Disclosure Information

 

Joe R. Monroe, MPAS, PA, ­practices at Dawkins ­Dermatology Clinic in Oklahoma City. He is also the founder of the Society of ­Dermatology ­Physician ­Assistants.

Article PDF
Article PDF

ANSWER
The correct answer is pityriasis rosea (choice “b”), a common and very distinctive eruption related to human herpesvirus 6 and 7.

Allergic reaction to methotrexate (choice “a”), while far from unknown, does not resemble pityriasis rosea. It also would not be limited to such a relatively small area.

Pityriasis rosea is often designated as “fungal infection” (choice “c”) by the uninitiated. However, the lesions of dermatophytosis would be round, with a leading scaly edge, and unlikely to be found in this distribution.

Secondary syphilis (choice “d”) is a major item in the pityriasis rosea differential, but it almost always involves the palms and soles and the lesions would be round (not oval) scaly brown papules. Furthermore, assuming we have an honest patient, we’re also missing a source for sexually transmitted infection.

DISCUSSION
One could hardly ask for a more classic case of pityriasis rosea (PR), which primarily affects patients ages 14 to 40. Alas, that being said, one cannot depend on seeing all these clues in every PR patient.

For example, the herald patch (also known as the mother patch) is missing in at least half of cases. In others, the lesions are smaller, sparser, and more papular (especially in young black patients). The condition may even be confined to intertriginous areas (eg, the groin and/or axillae); this is known as inverse PR.

While salmon-colored scaly lesions are considered a classic presentation, PR can present with darker ovoid macules that have minimal scale and, rarely, become bullous. Involvement above the neck is rare.

What is consistent and dependable among signs of PR is the centripetal scale, seen even in the smallest lesions. This scale is so fine that the old dermatology texts called it “cigarette paper” scale or “scurf.”

After decades of speculation, researchers finally provided strong evidence of the probable cause  of PR: replication of human herpesvirus 6 and 7, present in mono-
nuclear cells of lesional skin. Though universally acquired in childhood, these viruses are thought to remain latent until reactivated, leading to viremia.

Itching can be moderately severe in a minority of cases. Most patients, such as this one, are not bothered much by the condition once they understand its self-limited nature. They usually are not happy, however, to learn that it could persist for nine weeks or more, whether treated or not.

UV light exposure can be helpful in hastening PR’s departure, and topical corticosteroids (class III or IV; eg, triamcinolone 0.1% cream) can help control the itching. Neither oral nor topical antihistamines will help, since PR is not a histamine-mediated problem.

If the diagnosis is in doubt, a punch biopsy could at least rule out the more serious items in the differential, which include syphilis, drug rash, and psoriasis. In cases in which fungal origin is a possibility, a quick KOH prep will settle the issue. However, it must be remembered that one doesn’t just “get” a fungal infection. There has to be a source (animal, child), and that source is usually identified with minimal history taking.

ANSWER
The correct answer is pityriasis rosea (choice “b”), a common and very distinctive eruption related to human herpesvirus 6 and 7.

Allergic reaction to methotrexate (choice “a”), while far from unknown, does not resemble pityriasis rosea. It also would not be limited to such a relatively small area.

Pityriasis rosea is often designated as “fungal infection” (choice “c”) by the uninitiated. However, the lesions of dermatophytosis would be round, with a leading scaly edge, and unlikely to be found in this distribution.

Secondary syphilis (choice “d”) is a major item in the pityriasis rosea differential, but it almost always involves the palms and soles and the lesions would be round (not oval) scaly brown papules. Furthermore, assuming we have an honest patient, we’re also missing a source for sexually transmitted infection.

DISCUSSION
One could hardly ask for a more classic case of pityriasis rosea (PR), which primarily affects patients ages 14 to 40. Alas, that being said, one cannot depend on seeing all these clues in every PR patient.

For example, the herald patch (also known as the mother patch) is missing in at least half of cases. In others, the lesions are smaller, sparser, and more papular (especially in young black patients). The condition may even be confined to intertriginous areas (eg, the groin and/or axillae); this is known as inverse PR.

While salmon-colored scaly lesions are considered a classic presentation, PR can present with darker ovoid macules that have minimal scale and, rarely, become bullous. Involvement above the neck is rare.

What is consistent and dependable among signs of PR is the centripetal scale, seen even in the smallest lesions. This scale is so fine that the old dermatology texts called it “cigarette paper” scale or “scurf.”

After decades of speculation, researchers finally provided strong evidence of the probable cause  of PR: replication of human herpesvirus 6 and 7, present in mono-
nuclear cells of lesional skin. Though universally acquired in childhood, these viruses are thought to remain latent until reactivated, leading to viremia.

Itching can be moderately severe in a minority of cases. Most patients, such as this one, are not bothered much by the condition once they understand its self-limited nature. They usually are not happy, however, to learn that it could persist for nine weeks or more, whether treated or not.

UV light exposure can be helpful in hastening PR’s departure, and topical corticosteroids (class III or IV; eg, triamcinolone 0.1% cream) can help control the itching. Neither oral nor topical antihistamines will help, since PR is not a histamine-mediated problem.

If the diagnosis is in doubt, a punch biopsy could at least rule out the more serious items in the differential, which include syphilis, drug rash, and psoriasis. In cases in which fungal origin is a possibility, a quick KOH prep will settle the issue. However, it must be remembered that one doesn’t just “get” a fungal infection. There has to be a source (animal, child), and that source is usually identified with minimal history taking.

Issue
Clinician Reviews - 24(9)
Issue
Clinician Reviews - 24(9)
Page Number
11-12
Page Number
11-12
Publications
Publications
Topics
Article Type
Display Headline
Itchy Lesion Heralds Pervasive Problem
Display Headline
Itchy Lesion Heralds Pervasive Problem
Legacy Keywords
dermatology, dermadiagnosis, neck, chest, rash, allergic reaction, methotrexate, Pityriasis rosea, fungal infection, Secondary syphilis
Legacy Keywords
dermatology, dermadiagnosis, neck, chest, rash, allergic reaction, methotrexate, Pityriasis rosea, fungal infection, Secondary syphilis
Sections
Questionnaire Body

Patient with rash on chest and neck.
This itchy rash started on his back and spread to his chest and neck.

 

 

Two weeks ago, an itchy rash appeared on a man’s back before spreading to his chest and neck. He has never experienced anything like it before, and no one else in his household is similarly affected. He denies night sweats, fever, and malaise but reports that he was recently diagnosed with rheumatoid arthritis. His rheumatologist started him on methotrexate (12.5 mg/wk) after extensive labwork (complete blood count, complete metabolic panel, and hepatitis profile) was performed. He denies any history of high-risk sexual behavior or exposure, exposure to animals or children, or history of foreign travel. The patient, who appears well, is afebrile and in no distress. The original lesion, on his upper left back, is distinctly pinkish brown and round, with an odd fine scale around its inner rim, and measures about 3 cm in diameter. Elsewhere, examination reveals about 15 more lesions. All are oval but similarly pinkish brown, averaging about 2 cm in their long axis. These smaller lesions form a necklace-like configuration, paralleling the natural skin lines of the neck. Each lesion has central scaling identical to that of the original back lesion. Examination of the patient’s palms and soles fails to reveal any cutaneous abnormalities. Likewise, examination of the oral cavity is normal. No palpable nodes are felt around the neck, in the axillae, or in the groin.

 

Disallow All Ads
Article PDF Media

Intergluteal Itching in Need of Relief

Article Type
Changed
Tue, 12/13/2016 - 12:08
Display Headline
Intergluteal Itching in Need of Relief

 

ANSWER

 

Admittedly, this is a bit of a trick question—but with a good teaching point to make. A course of oral fluconazole (choice “a”) is futile, since there’s no reason to think this problem is yeast-driven and since the patient has already demonstrated a lack of response to topical imidazoles.

Punch biopsy (choice “b”) would be a good choice, but not in this area, where it could quickly become a bigger problem than the one the patient presented with. Sutures would not likely hold the biopsy wound together, and resultant infection is all too likely.

A KOH test to detect fungal or yeast elements (choice “c”) is unlikely to shed any light on the problem, given the lack of response to antifungal creams. Finally, there’s no reason to suspect a bacterial origin, so oral antibiotics such as cephalexin (choice “d”) would be useless (and had already been tried unsuccessfully).

The correct answer is none of the above (choice “e”).

DISCUSSION

This case illustrates why dermatology seems so maddeningly difficult to the uninitiated. Any experienced derm provider would know the correct diagnosis, lichen sclerosus et atrophicus (LS&A), because it presents in such a distinctive way (in limited locations, predominantly in women) and because the differential is so limited. But if you’ve never heard of LS&A, you’re unlikely to diagnose it, let alone know how to treat it.

LS&A is an inflammatory condition of unknown origin that affects the upper epidermis. It can present in extragenital locations (particularly shoulders and legs) but is far more common in genital areas. As exhibited in this case, it presents with well-defined pigment loss, which is especially easy to see in patients with darker skin.

Although more commonly seen in women, LS&A can occur in men, usually manifesting on the penile glans and distal foreskin of uncircumcised patients. The dry atrophic changes seen on the glans can lead to stenosis of the urethral meatus and, proximally, to adhesions (phimosis) of the foreskin. (This condition was termed balanitis xerotica obliterans [BXO] long before its pathologic process was determined to be identical to LS&A’s. Tissue specimens obtained during circumcisions performed for chronic phimosis often yield evidence of BXO.)

In women, untreated chronic LS&A can lead to sclerotic changes in and around the urethra and labia minora and can cause introital stenosis. This case is a bit atypical; LS&A more often manifests in perivaginal and perirectal areas, where the intense hypopigmentation produces a classic “figure eight” appearance.

The differential includes lichen simplex chronicus, psoriasis, lichen planus, contact/irritant dermatitis, and seborrhea. Often, biopsy is necessary and appropriate to settle the issue, other factors being equal.

TREATMENT/PROGNOSIS

The patient was given a prescription for clobetasol 0.05% ointment for twice-daily application Monday through Friday (and no application for two consecutive days—in this case, the weekend—per week). Studies have established the efficacy and safety of this treatment regimen.

In a month or two, application can be reduced to once or twice a week to control the condition.

Article PDF
Author and Disclosure Information

 

Joe R. Monroe, MPAS, PA, ­practices at Dawkins ­Dermatology Clinic in Oklahoma City. He is also the founder of the Society of ­Dermatology ­Physician ­Assistants.

Issue
Clinician Reviews - 24(8)
Publications
Topics
Page Number
14-15
Legacy Keywords
lichen sclerosus et atrophicus, LS&A, Dermadiagnosis, derm, itchy, perianal, yeast, psoriasis, punch biopsy, koh prep,
Sections
Author and Disclosure Information

 

Joe R. Monroe, MPAS, PA, ­practices at Dawkins ­Dermatology Clinic in Oklahoma City. He is also the founder of the Society of ­Dermatology ­Physician ­Assistants.

Author and Disclosure Information

 

Joe R. Monroe, MPAS, PA, ­practices at Dawkins ­Dermatology Clinic in Oklahoma City. He is also the founder of the Society of ­Dermatology ­Physician ­Assistants.

Article PDF
Article PDF

 

ANSWER

 

Admittedly, this is a bit of a trick question—but with a good teaching point to make. A course of oral fluconazole (choice “a”) is futile, since there’s no reason to think this problem is yeast-driven and since the patient has already demonstrated a lack of response to topical imidazoles.

Punch biopsy (choice “b”) would be a good choice, but not in this area, where it could quickly become a bigger problem than the one the patient presented with. Sutures would not likely hold the biopsy wound together, and resultant infection is all too likely.

A KOH test to detect fungal or yeast elements (choice “c”) is unlikely to shed any light on the problem, given the lack of response to antifungal creams. Finally, there’s no reason to suspect a bacterial origin, so oral antibiotics such as cephalexin (choice “d”) would be useless (and had already been tried unsuccessfully).

The correct answer is none of the above (choice “e”).

DISCUSSION

This case illustrates why dermatology seems so maddeningly difficult to the uninitiated. Any experienced derm provider would know the correct diagnosis, lichen sclerosus et atrophicus (LS&A), because it presents in such a distinctive way (in limited locations, predominantly in women) and because the differential is so limited. But if you’ve never heard of LS&A, you’re unlikely to diagnose it, let alone know how to treat it.

LS&A is an inflammatory condition of unknown origin that affects the upper epidermis. It can present in extragenital locations (particularly shoulders and legs) but is far more common in genital areas. As exhibited in this case, it presents with well-defined pigment loss, which is especially easy to see in patients with darker skin.

Although more commonly seen in women, LS&A can occur in men, usually manifesting on the penile glans and distal foreskin of uncircumcised patients. The dry atrophic changes seen on the glans can lead to stenosis of the urethral meatus and, proximally, to adhesions (phimosis) of the foreskin. (This condition was termed balanitis xerotica obliterans [BXO] long before its pathologic process was determined to be identical to LS&A’s. Tissue specimens obtained during circumcisions performed for chronic phimosis often yield evidence of BXO.)

In women, untreated chronic LS&A can lead to sclerotic changes in and around the urethra and labia minora and can cause introital stenosis. This case is a bit atypical; LS&A more often manifests in perivaginal and perirectal areas, where the intense hypopigmentation produces a classic “figure eight” appearance.

The differential includes lichen simplex chronicus, psoriasis, lichen planus, contact/irritant dermatitis, and seborrhea. Often, biopsy is necessary and appropriate to settle the issue, other factors being equal.

TREATMENT/PROGNOSIS

The patient was given a prescription for clobetasol 0.05% ointment for twice-daily application Monday through Friday (and no application for two consecutive days—in this case, the weekend—per week). Studies have established the efficacy and safety of this treatment regimen.

In a month or two, application can be reduced to once or twice a week to control the condition.

 

ANSWER

 

Admittedly, this is a bit of a trick question—but with a good teaching point to make. A course of oral fluconazole (choice “a”) is futile, since there’s no reason to think this problem is yeast-driven and since the patient has already demonstrated a lack of response to topical imidazoles.

Punch biopsy (choice “b”) would be a good choice, but not in this area, where it could quickly become a bigger problem than the one the patient presented with. Sutures would not likely hold the biopsy wound together, and resultant infection is all too likely.

A KOH test to detect fungal or yeast elements (choice “c”) is unlikely to shed any light on the problem, given the lack of response to antifungal creams. Finally, there’s no reason to suspect a bacterial origin, so oral antibiotics such as cephalexin (choice “d”) would be useless (and had already been tried unsuccessfully).

The correct answer is none of the above (choice “e”).

DISCUSSION

This case illustrates why dermatology seems so maddeningly difficult to the uninitiated. Any experienced derm provider would know the correct diagnosis, lichen sclerosus et atrophicus (LS&A), because it presents in such a distinctive way (in limited locations, predominantly in women) and because the differential is so limited. But if you’ve never heard of LS&A, you’re unlikely to diagnose it, let alone know how to treat it.

LS&A is an inflammatory condition of unknown origin that affects the upper epidermis. It can present in extragenital locations (particularly shoulders and legs) but is far more common in genital areas. As exhibited in this case, it presents with well-defined pigment loss, which is especially easy to see in patients with darker skin.

Although more commonly seen in women, LS&A can occur in men, usually manifesting on the penile glans and distal foreskin of uncircumcised patients. The dry atrophic changes seen on the glans can lead to stenosis of the urethral meatus and, proximally, to adhesions (phimosis) of the foreskin. (This condition was termed balanitis xerotica obliterans [BXO] long before its pathologic process was determined to be identical to LS&A’s. Tissue specimens obtained during circumcisions performed for chronic phimosis often yield evidence of BXO.)

In women, untreated chronic LS&A can lead to sclerotic changes in and around the urethra and labia minora and can cause introital stenosis. This case is a bit atypical; LS&A more often manifests in perivaginal and perirectal areas, where the intense hypopigmentation produces a classic “figure eight” appearance.

The differential includes lichen simplex chronicus, psoriasis, lichen planus, contact/irritant dermatitis, and seborrhea. Often, biopsy is necessary and appropriate to settle the issue, other factors being equal.

TREATMENT/PROGNOSIS

The patient was given a prescription for clobetasol 0.05% ointment for twice-daily application Monday through Friday (and no application for two consecutive days—in this case, the weekend—per week). Studies have established the efficacy and safety of this treatment regimen.

In a month or two, application can be reduced to once or twice a week to control the condition.

Issue
Clinician Reviews - 24(8)
Issue
Clinician Reviews - 24(8)
Page Number
14-15
Page Number
14-15
Publications
Publications
Topics
Article Type
Display Headline
Intergluteal Itching in Need of Relief
Display Headline
Intergluteal Itching in Need of Relief
Legacy Keywords
lichen sclerosus et atrophicus, LS&A, Dermadiagnosis, derm, itchy, perianal, yeast, psoriasis, punch biopsy, koh prep,
Legacy Keywords
lichen sclerosus et atrophicus, LS&A, Dermadiagnosis, derm, itchy, perianal, yeast, psoriasis, punch biopsy, koh prep,
Sections
Questionnaire Body

 

 

For almost a year, a 55-year-old African-American woman has experienced itchy skin changes in her perianal area. Treatment attempts with several topical creams—including clotrimazole, combination clotrimazole/betamethasone, and ketoconazole—have not helped. The patient has seen several primary care providers for the problem. All have told her that it was yeast-related, except the last clinician, who suspected psoriasis. When the topical medication prescribed by that provider did not yield a resolution, the patient decided to consult dermatology. Due to her lack of insurance, she had to wait four months to see a derm clinician, since her only option was a once-a-month free clinic in her community. Aside from mild hypertension, the patient claims to be in good health. Recent work-up indicated she does not have diabetes. She denies any family history of skin diseases, including psoriasis. She has had no previous complaints regarding her vaginal/perivaginal areas. The patient’s type V skin is free of notable changes except in the intergluteal and perianal areas. Specifically, no rash is noted on her extensor elbows or knees or in her scalp, and there are no changes in her fingernails. When the patient lies on her left side, extending her left leg and bringing her right knee toward her chest, the entire intergluteal and perianal areas can be visualized. Distinct loss of dark pigment is seen in the upper intergluteal/lower coccygeal areas. Closer inspection reveals that the pigment loss is complete, giving the affected skin a porcelain-like white appearance that also seems moderately atrophic. Palpation confirms this impression. No such changes are noted in the perianal or perineal areas. However, there is diffuse hyperpigmentation, as well as signs of mild chronic excoriation.

 

Disallow All Ads
Article PDF Media

How Are Lesions and Seizures Related?

Article Type
Changed
Thu, 09/20/2018 - 09:51
Display Headline
How Are Lesions and Seizures Related?

ANSWER
The correct answer is tuberous sclerosis (choice “d”), a genetic disorder affecting cellular differentiation and proliferation. As a result, hamartomas commonly form in organs such as the brain.

Von Recklinghausen disease (choice “a”; also known as neurofibromatosis type 1) presents with hyperpigmented macules and patches called café au lait macules. These and other findings differentiate it considerably from tuberous sclerosis.

“No unifying explanation” (choice “b”) is incorrect, because, as stated above, there is a unifying explanation for these findings!

Vitiligo (choice “c”) involves hypopigmented macules and patches. However, it does not present with some of the other findings seen in this case.

DISCUSSION
Tuberous sclerosis was first described in 1862 by von Recklinghausen, who noted the collection of findings we now term tuberous sclerosis complex (TSC) due to the wide variation in presentation.

In the United States, TSC occurs in an estimated one in 6,000 to one in 30,000 newborns. Usually, it manifests when the child is between ages 2 and 6, but genetic penetrance is highly variable—a major reason for the shift to the use of the term complex with this condition. (This case, with its late presentation, illustrates this variability.)

In about 60% of cases, TSC is transmitted in autosomal dominant mode, although at least 20% of cases may be due to spontaneous mutations. It has not shown any racial or gender predilections. Up to 90% of TSC patients will present with seizures, and 60% to 70% of those affected will demonstrate some developmental disability.

Skin is affected in at least 70% of TSC cases. This patient typifies the common dermatologic findings: dart-shaped hypopigmented macules (known as mountain ash leaf spots) and fleshy facial papules in the nasolabial region (originally called adenoma sebaceum and now called angiofibromas). Periungal fibromas are also common. Other major diagnostic criteria for TSC include cortical tubers seen on imaging and retinal hamartomas. Minor diagnostic criteria include pits in dental enamel and gingival fibromas.

Definitive diagnosis depends on identification of either two major manifestations or one major and two minor findings. There are no blood tests to confirm the diagnosis. In terms of treatment, the various elements that comprise TSC can be dealt with (eg, control of seizures and destruction or modification of skin lesions).

This patient received a diagnosis of TSC. She was returned to her primary care provider, who referred her for additional imaging of her lungs and heart and for ophthalmologic evaluation of her retinas. Genetic counseling was also arranged. With her seizures under control, she was able to continue matriculation through high school.

Article PDF
Author and Disclosure Information

Joe R. Monroe, MPAS, PA, ­practices at Dawkins ­Dermatology Clinic in Oklahoma City. He is also the founder of the Society of ­Dermatology ­Physician ­Assistants.

Issue
Clinician Reviews - 24(7)
Publications
Topics
Page Number
11-12
Legacy Keywords
Dermadiagnosis, von Recklinghausen disease, Vitiligo, Tuberous sclerosis, hamartomas, tuberous sclerosis complex, lesions, pediatrics, grand mal seizure, seizures
Sections
Author and Disclosure Information

Joe R. Monroe, MPAS, PA, ­practices at Dawkins ­Dermatology Clinic in Oklahoma City. He is also the founder of the Society of ­Dermatology ­Physician ­Assistants.

Author and Disclosure Information

Joe R. Monroe, MPAS, PA, ­practices at Dawkins ­Dermatology Clinic in Oklahoma City. He is also the founder of the Society of ­Dermatology ­Physician ­Assistants.

Article PDF
Article PDF

ANSWER
The correct answer is tuberous sclerosis (choice “d”), a genetic disorder affecting cellular differentiation and proliferation. As a result, hamartomas commonly form in organs such as the brain.

Von Recklinghausen disease (choice “a”; also known as neurofibromatosis type 1) presents with hyperpigmented macules and patches called café au lait macules. These and other findings differentiate it considerably from tuberous sclerosis.

“No unifying explanation” (choice “b”) is incorrect, because, as stated above, there is a unifying explanation for these findings!

Vitiligo (choice “c”) involves hypopigmented macules and patches. However, it does not present with some of the other findings seen in this case.

DISCUSSION
Tuberous sclerosis was first described in 1862 by von Recklinghausen, who noted the collection of findings we now term tuberous sclerosis complex (TSC) due to the wide variation in presentation.

In the United States, TSC occurs in an estimated one in 6,000 to one in 30,000 newborns. Usually, it manifests when the child is between ages 2 and 6, but genetic penetrance is highly variable—a major reason for the shift to the use of the term complex with this condition. (This case, with its late presentation, illustrates this variability.)

In about 60% of cases, TSC is transmitted in autosomal dominant mode, although at least 20% of cases may be due to spontaneous mutations. It has not shown any racial or gender predilections. Up to 90% of TSC patients will present with seizures, and 60% to 70% of those affected will demonstrate some developmental disability.

Skin is affected in at least 70% of TSC cases. This patient typifies the common dermatologic findings: dart-shaped hypopigmented macules (known as mountain ash leaf spots) and fleshy facial papules in the nasolabial region (originally called adenoma sebaceum and now called angiofibromas). Periungal fibromas are also common. Other major diagnostic criteria for TSC include cortical tubers seen on imaging and retinal hamartomas. Minor diagnostic criteria include pits in dental enamel and gingival fibromas.

Definitive diagnosis depends on identification of either two major manifestations or one major and two minor findings. There are no blood tests to confirm the diagnosis. In terms of treatment, the various elements that comprise TSC can be dealt with (eg, control of seizures and destruction or modification of skin lesions).

This patient received a diagnosis of TSC. She was returned to her primary care provider, who referred her for additional imaging of her lungs and heart and for ophthalmologic evaluation of her retinas. Genetic counseling was also arranged. With her seizures under control, she was able to continue matriculation through high school.

ANSWER
The correct answer is tuberous sclerosis (choice “d”), a genetic disorder affecting cellular differentiation and proliferation. As a result, hamartomas commonly form in organs such as the brain.

Von Recklinghausen disease (choice “a”; also known as neurofibromatosis type 1) presents with hyperpigmented macules and patches called café au lait macules. These and other findings differentiate it considerably from tuberous sclerosis.

“No unifying explanation” (choice “b”) is incorrect, because, as stated above, there is a unifying explanation for these findings!

Vitiligo (choice “c”) involves hypopigmented macules and patches. However, it does not present with some of the other findings seen in this case.

DISCUSSION
Tuberous sclerosis was first described in 1862 by von Recklinghausen, who noted the collection of findings we now term tuberous sclerosis complex (TSC) due to the wide variation in presentation.

In the United States, TSC occurs in an estimated one in 6,000 to one in 30,000 newborns. Usually, it manifests when the child is between ages 2 and 6, but genetic penetrance is highly variable—a major reason for the shift to the use of the term complex with this condition. (This case, with its late presentation, illustrates this variability.)

In about 60% of cases, TSC is transmitted in autosomal dominant mode, although at least 20% of cases may be due to spontaneous mutations. It has not shown any racial or gender predilections. Up to 90% of TSC patients will present with seizures, and 60% to 70% of those affected will demonstrate some developmental disability.

Skin is affected in at least 70% of TSC cases. This patient typifies the common dermatologic findings: dart-shaped hypopigmented macules (known as mountain ash leaf spots) and fleshy facial papules in the nasolabial region (originally called adenoma sebaceum and now called angiofibromas). Periungal fibromas are also common. Other major diagnostic criteria for TSC include cortical tubers seen on imaging and retinal hamartomas. Minor diagnostic criteria include pits in dental enamel and gingival fibromas.

Definitive diagnosis depends on identification of either two major manifestations or one major and two minor findings. There are no blood tests to confirm the diagnosis. In terms of treatment, the various elements that comprise TSC can be dealt with (eg, control of seizures and destruction or modification of skin lesions).

This patient received a diagnosis of TSC. She was returned to her primary care provider, who referred her for additional imaging of her lungs and heart and for ophthalmologic evaluation of her retinas. Genetic counseling was also arranged. With her seizures under control, she was able to continue matriculation through high school.

Issue
Clinician Reviews - 24(7)
Issue
Clinician Reviews - 24(7)
Page Number
11-12
Page Number
11-12
Publications
Publications
Topics
Article Type
Display Headline
How Are Lesions and Seizures Related?
Display Headline
How Are Lesions and Seizures Related?
Legacy Keywords
Dermadiagnosis, von Recklinghausen disease, Vitiligo, Tuberous sclerosis, hamartomas, tuberous sclerosis complex, lesions, pediatrics, grand mal seizure, seizures
Legacy Keywords
Dermadiagnosis, von Recklinghausen disease, Vitiligo, Tuberous sclerosis, hamartomas, tuberous sclerosis complex, lesions, pediatrics, grand mal seizure, seizures
Sections
Questionnaire Body

 

 

The parents of this 16-year-old girl first noticed the “spots” on their daughter’s thigh when she was 2. At that time, they expressed mild concern to the child’s pediatrician, who advised them to watch the lesions for change. In the absence of any, the spots were essentially ignored. But recently—and alarmingly—the patient experienced two grand mal seizures. Three months ago, she was hospitalized and underwent a thorough examination and workup, including imaging studies of the brain. These revealed presumed neural tumors, which are being followed with serial imaging. In the meantime, her primary care provider recommends a visit to dermatology for evaluation of the child’s skin—including the aforementioned spots on her thigh. Her parents deny any family history of similar problems with skin or seizures. The spots are located on the patient’s right lateral thigh. The largest is a dart-shaped 4 x 2.5–cm hypopigmented patch. It is surrounded by much smaller (< 1 cm in diameter) but similarly hypopigmented macules. The large lesion is symmetrical but has slightly serrated borders. Examination elsewhere reveals periungual fibromas on two of 10 fingers. Odd fleshy papules are noted in the bilateral nasolabial areas.

 

Disallow All Ads
Alternative CME
Use ProPublica
Article PDF Media

Man Is Alarmed by Skin Lesions

Article Type
Changed
Tue, 12/13/2016 - 12:08
Display Headline
Man Is Alarmed by Skin Lesions

ANSWER

The correct answer is eruptive xanthomata (choice “b”) caused by an accumulation of lipid-filled macrophages as a result of pathologic levels of serum triglyceride—a situation discussed more fully below.

Neurofibromatosis type I (choice “a”), also known as von Recklinghausen disease, can present with multiple intradermal nodules. However, it usually appears in the second or third decade of life, with lesions that are fixed and soft. Biopsy would have confirmed this diagnosis.

Diabetic dermopathy (choice “c”) manifests with atrophic patches on anterior tibial skin. The patches occasionally become superficially eroded but do not resemble this patient’s lesions at all.

Juvenile xanthogranuloma (choice “d”) usually presents on children as a solitary yellowish brown papule. It can resemble eruptive xanthomata histologically but not clinically.

DISCUSSION

Eruptive xanthomata (EX) are relatively common, manifesting rapidly as papules and nodules, most frequently in the setting of hypertriglyceridemia. The latter can be familial and may be worsened by poorly controlled diabetes. Persons with Fredrickson types I, IV, and V hyperlipidemia are especially prone to EX.

As might be expected, patients with EX are at risk for several associated morbidities, including acute pancreatitis (especially in childhood cases) and atherosclerotic vessel disease. EX have also been associated with hypothyroid states and nephrotic syndromes.

Elevations in cholesterol, with normal triglyceride levels, can be associated with several types of xanthoma, including plane xanthomas and xanthelasma. The latter, often benign, can manifest in a normolipemic patient as well (necessitating a problem-directed history, physical, and lipid check).

Biopsy is often required to confirm the diagnosis of EX. As in this case, it typically shows monotonous collections of lipid-laden macrophages. Frozen sections of EX can be successfully stained for lipids, but routine processing of specimens effectively removes any lipids, replacing them with paraffin.

TREATMENT

Treatment entails controlling lipids with medication (fenofibrate), diet, and exercise and getting diabetes under control, as indicated. It is also essential to assess for atherosclerotic vessel disease and rule out pancreatitis.

Within a month of institution of treatment, this patient’s lesions had all but disappeared. His serum amylase and lipase were within normal limits, and testing for atherosclerotic vessel disease was pending.

Click here for more DermaDiagnosis cases, including 
• The Value of Certainty in Diagnosis
• A Purplish Rash on the Instep
• Hair Loss at a Very Young Age.

Author and Disclosure Information

 

Joe R. Monroe, MPAS, PA, ­practices at Dawkins ­Dermatology Clinic in Oklahoma City. He is also the founder of the Society of ­Dermatology ­Physician ­Assistants.

Issue
Clinician Reviews - 24(6)
Publications
Topics
Page Number
12-13
Legacy Keywords
Dermadiagnosis, derm, Neurofibromatosis type I, Eruptive xanthomata, Diabetic dermopathy, Juvenile xanthogranuloma, diabetes, intradermal papules, nodules, lesions, macrophages, serum triglyceride
Sections
Author and Disclosure Information

 

Joe R. Monroe, MPAS, PA, ­practices at Dawkins ­Dermatology Clinic in Oklahoma City. He is also the founder of the Society of ­Dermatology ­Physician ­Assistants.

Author and Disclosure Information

 

Joe R. Monroe, MPAS, PA, ­practices at Dawkins ­Dermatology Clinic in Oklahoma City. He is also the founder of the Society of ­Dermatology ­Physician ­Assistants.

Related Articles

ANSWER

The correct answer is eruptive xanthomata (choice “b”) caused by an accumulation of lipid-filled macrophages as a result of pathologic levels of serum triglyceride—a situation discussed more fully below.

Neurofibromatosis type I (choice “a”), also known as von Recklinghausen disease, can present with multiple intradermal nodules. However, it usually appears in the second or third decade of life, with lesions that are fixed and soft. Biopsy would have confirmed this diagnosis.

Diabetic dermopathy (choice “c”) manifests with atrophic patches on anterior tibial skin. The patches occasionally become superficially eroded but do not resemble this patient’s lesions at all.

Juvenile xanthogranuloma (choice “d”) usually presents on children as a solitary yellowish brown papule. It can resemble eruptive xanthomata histologically but not clinically.

DISCUSSION

Eruptive xanthomata (EX) are relatively common, manifesting rapidly as papules and nodules, most frequently in the setting of hypertriglyceridemia. The latter can be familial and may be worsened by poorly controlled diabetes. Persons with Fredrickson types I, IV, and V hyperlipidemia are especially prone to EX.

As might be expected, patients with EX are at risk for several associated morbidities, including acute pancreatitis (especially in childhood cases) and atherosclerotic vessel disease. EX have also been associated with hypothyroid states and nephrotic syndromes.

Elevations in cholesterol, with normal triglyceride levels, can be associated with several types of xanthoma, including plane xanthomas and xanthelasma. The latter, often benign, can manifest in a normolipemic patient as well (necessitating a problem-directed history, physical, and lipid check).

Biopsy is often required to confirm the diagnosis of EX. As in this case, it typically shows monotonous collections of lipid-laden macrophages. Frozen sections of EX can be successfully stained for lipids, but routine processing of specimens effectively removes any lipids, replacing them with paraffin.

TREATMENT

Treatment entails controlling lipids with medication (fenofibrate), diet, and exercise and getting diabetes under control, as indicated. It is also essential to assess for atherosclerotic vessel disease and rule out pancreatitis.

Within a month of institution of treatment, this patient’s lesions had all but disappeared. His serum amylase and lipase were within normal limits, and testing for atherosclerotic vessel disease was pending.

Click here for more DermaDiagnosis cases, including 
• The Value of Certainty in Diagnosis
• A Purplish Rash on the Instep
• Hair Loss at a Very Young Age.

ANSWER

The correct answer is eruptive xanthomata (choice “b”) caused by an accumulation of lipid-filled macrophages as a result of pathologic levels of serum triglyceride—a situation discussed more fully below.

Neurofibromatosis type I (choice “a”), also known as von Recklinghausen disease, can present with multiple intradermal nodules. However, it usually appears in the second or third decade of life, with lesions that are fixed and soft. Biopsy would have confirmed this diagnosis.

Diabetic dermopathy (choice “c”) manifests with atrophic patches on anterior tibial skin. The patches occasionally become superficially eroded but do not resemble this patient’s lesions at all.

Juvenile xanthogranuloma (choice “d”) usually presents on children as a solitary yellowish brown papule. It can resemble eruptive xanthomata histologically but not clinically.

DISCUSSION

Eruptive xanthomata (EX) are relatively common, manifesting rapidly as papules and nodules, most frequently in the setting of hypertriglyceridemia. The latter can be familial and may be worsened by poorly controlled diabetes. Persons with Fredrickson types I, IV, and V hyperlipidemia are especially prone to EX.

As might be expected, patients with EX are at risk for several associated morbidities, including acute pancreatitis (especially in childhood cases) and atherosclerotic vessel disease. EX have also been associated with hypothyroid states and nephrotic syndromes.

Elevations in cholesterol, with normal triglyceride levels, can be associated with several types of xanthoma, including plane xanthomas and xanthelasma. The latter, often benign, can manifest in a normolipemic patient as well (necessitating a problem-directed history, physical, and lipid check).

Biopsy is often required to confirm the diagnosis of EX. As in this case, it typically shows monotonous collections of lipid-laden macrophages. Frozen sections of EX can be successfully stained for lipids, but routine processing of specimens effectively removes any lipids, replacing them with paraffin.

TREATMENT

Treatment entails controlling lipids with medication (fenofibrate), diet, and exercise and getting diabetes under control, as indicated. It is also essential to assess for atherosclerotic vessel disease and rule out pancreatitis.

Within a month of institution of treatment, this patient’s lesions had all but disappeared. His serum amylase and lipase were within normal limits, and testing for atherosclerotic vessel disease was pending.

Click here for more DermaDiagnosis cases, including 
• The Value of Certainty in Diagnosis
• A Purplish Rash on the Instep
• Hair Loss at a Very Young Age.

Issue
Clinician Reviews - 24(6)
Issue
Clinician Reviews - 24(6)
Page Number
12-13
Page Number
12-13
Publications
Publications
Topics
Article Type
Display Headline
Man Is Alarmed by Skin Lesions
Display Headline
Man Is Alarmed by Skin Lesions
Legacy Keywords
Dermadiagnosis, derm, Neurofibromatosis type I, Eruptive xanthomata, Diabetic dermopathy, Juvenile xanthogranuloma, diabetes, intradermal papules, nodules, lesions, macrophages, serum triglyceride
Legacy Keywords
Dermadiagnosis, derm, Neurofibromatosis type I, Eruptive xanthomata, Diabetic dermopathy, Juvenile xanthogranuloma, diabetes, intradermal papules, nodules, lesions, macrophages, serum triglyceride
Sections
Questionnaire Body

 

 

Although they are unaccompanied by any other symptoms, this man is understandably alarmed by the extensive lesions covering much of his body. They first appeared months ago but have become more numerous, larger, and more prominent with time. The patient’s history includes type 2 diabetes (often poorly controlled) and dyslipidemia, for which he takes fenofibrate. Several years ago, he experienced a similar skin outbreak, which resolved after the patient increased his exercise and gained better control of his blood glucose. The condition is striking. There are widespread bilateral collections of shallow intradermal papules, nodules, and plaques primarily on the extensor surfaces of the patient’s arms, legs, and thighs and the convex surfaces of his buttocks. Numbering into the hundreds, the lesions spare his palms, soles, face, and scalp. No abnormality of the periocular skin is appreciated. Moderately firm on palpation, the lesions range in size from 1 to 3 cm in diameter. In several locations, they are linearly configured. A 4-mm punch biopsy of one of them shows large numbers of foamy macrophages in the epidermis and upper dermis. Bloodwork reveals a triglyceride level of 3,850 mg/dL.

 

Disallow All Ads